选H时肯定得选K; 若她选择J,则她肯定选择W; 若她选择K,则她肯定选择X。 若该养鱼爱好者选择了Y,则下面哪一项是她可能选择的鱼群?

admin2014-11-05  19

问题         选H时肯定得选K;
    若她选择J,则她肯定选择W;
    若她选择K,则她肯定选择X。
若该养鱼爱好者选择了Y,则下面哪一项是她可能选择的鱼群?

选项 A、G,H,K
B、H,J,K
C、H,J,L
D、H,K,L

答案D

解析 选择了y就不能选择G;J和K不能同时被选,因为这样将迫使她选择3种水草,所以H和L就必须同时被选,根据条件(2)可知H被选时,肯定得选K而不能选择J,所以她可以选择的3种鱼是H,K和L。
转载请注明原文地址:https://jikaoti.com/ti/u3RjFFFM
0

相关试题推荐
随机试题
最新回复(0)